in millington experts pls reply

This topic has expert replies
Senior | Next Rank: 100 Posts
Posts: 51
Joined: Wed Feb 17, 2010 2:00 am
Thanked: 2 times

in millington experts pls reply

by rahul_tgsp » Thu Apr 14, 2011 10:21 pm
In Millington, a city of 50,000 people, Mercedes Pedrosa, a realtor, calculated that a fimily with Millington's median family income,$28,000 a year, could afford to buy Millington's median-priced $77,000 house. This calculation was based of an 11.2 percent mortgage interest rate and on the realtor's assumption that a family culd only afford to pay up a 25% of its income for housing.

Which of the following correctins of a fugure appearing in the passage above, fir it were the only correction that needed to be made, would yield a new calculation showing that even below the median family income would enable families in Millington to afford Millington's median- priced house?

A. Millington's total population was 45,000 people
B. Millington's median annual family income was $27,000
C. Millington's median-priced house cost $80,000
D. The rate at which piople in Millington had to pay mortgage interest was only 10%
E. Families in Millington could only afford to pay up to 22% of their annual income for housing
[spoiler]got to the ans but rn't really happy[/spoiler]

source unsure

User avatar
Legendary Member
Posts: 1101
Joined: Fri Jan 28, 2011 7:26 am
Thanked: 47 times
Followed by:13 members
GMAT Score:640

by HSPA » Thu Apr 14, 2011 11:02 pm
25% of income = 7K

7k for 11 months will come to = 77K and 11% interest on 84K is ~9K Now total 86K

If rate of interest can go down even a 27K guy can afford 86K

D i guess
First take: 640 (50M, 27V) - RC needs 300% improvement
Second take: coming soon..
Regards,
HSPA.

User avatar
Legendary Member
Posts: 582
Joined: Tue Mar 08, 2011 12:48 am
Thanked: 61 times
Followed by:6 members
GMAT Score:740

by force5 » Thu Apr 14, 2011 11:10 pm
D is the best of the lot.

User avatar
Legendary Member
Posts: 979
Joined: Tue Apr 14, 2009 1:38 am
Location: Hyderabad, India
Thanked: 49 times
Followed by:12 members
GMAT Score:700

by bubbliiiiiiii » Thu Apr 14, 2011 11:42 pm
rahul_tgsp wrote:In Millington, a city of 50,000 people, Mercedes Pedrosa, a realtor, calculated that a fimily with Millington's median family income,$28,000 a year, could afford to buy Millington's median-priced $77,000 house. This calculation was based of an 11.2 percent mortgage interest rate and on the realtor's assumption that a family culd only afford to pay up a 25% of its income for housing.

Which of the following correctins of a fugure appearing in the passage above, fir it were the only correction that needed to be made, would yield a new calculation showing that even below the median family income would enable families in Millington to afford Millington's median- priced house?

A. Millington's total population was 45,000 people
Population and its effects on housing is not discussed. Thus out of scope.
B. Millington's median annual family income was $27,000
This option disables the median income family from owning a house.
C. Millington's median-priced house cost $80,000
Same as B.
D. The rate at which piople in Millington had to pay mortgage interest was only 10%
This option talks that if the rate of interest is reduced the overall costing of the house would be reduced making it much cheaper to buy. Thus correct.
E. Families in Millington could only afford to pay up to 22% of their annual income for housing
Same as B.

[spoiler]got to the ans but rn't really happy[/spoiler]

source unsure
Regards,

Pranay

User avatar
Legendary Member
Posts: 1255
Joined: Fri Nov 07, 2008 2:08 pm
Location: St. Louis
Thanked: 312 times
Followed by:90 members

by Tani » Fri Apr 15, 2011 1:43 pm
To start with, the initial calculation is off. 11.2% of $77,000 is $8,624 (interest only, doesn't assume any amortization, which is, of course, incorrect, and which would raise the cost). $8,624 is 25% of $34,500, not $28,000, so the $28,000 family is out of the market.

However, to lower the income needed you have to lower the cost of the mortgage.

A - changes the population - that doesn't affect the mortgage cost.
B lowers the income - that makes it less likely that someone could afford the media-priced house.
C raises the price of the house - again, making it less likely.
E lowers the amount a family could afford to pay, again making it less likely.

D lowers the interest rate and therefore the cost. However, the poor family is still looking at an interest cost alone of $7,700, which is 25% of $30,800, so the $28,000 income family is still out of luck.

Where did this question come from?
Tani Wolff

User avatar
Legendary Member
Posts: 582
Joined: Tue Mar 08, 2011 12:48 am
Thanked: 61 times
Followed by:6 members
GMAT Score:740

by force5 » Sat Apr 16, 2011 3:00 am
Grt explanation Ms Wolff i thought on the same lines too. Was not too convinced with the choices.

Senior | Next Rank: 100 Posts
Posts: 51
Joined: Wed Feb 17, 2010 2:00 am
Thanked: 2 times

by rahul_tgsp » Sat Apr 16, 2011 8:26 am
thanks a lot
i too had picked D however the ans was confusing in many terms

@Tani still dunno abt the source